2024 AMC 10A Problems/Problem 16

Revision as of 21:28, 20 March 2025 by Maa is stupid (talk | contribs)

Problem

In how many ways can the integers $1$, $2$, $3$, $4$, $5$, and $6$ be arranged in a line so that the following statement is true? If $2$ is not adjacent to $3$, then $3$ is not adjacent to $4$.

$\textbf{(A)}~480 \qquad\textbf{(B)}~504 \qquad\textbf{(C)}~528 \qquad\textbf{(D)}~572 \qquad\textbf{(E)}~600$

Solution

We take the contrapositive, where the statement becomes "If $3$ is adjacent to $4$, then $2$ is adjacent to $3$."

If $3$ is adjacent to $4$, then $2$ is also adjacent to $3$ so we can group the three numbers together in two ways: $234$ or $432$. Then, the arrangements consists of this block of numbers and then three other numbers, which can be ordered in $4!$ ways. Hence, in this case, the total count is $2 \cdot 4! = 48$. If $3$ is not adjacent to $4$, there are no restrictions, so we can place $3$ and $4$ in $2 \cdot \tbinom{5}{2} = 20$ ways and then place the rest in $4! = 24$ ways, for a total of $480$ configurations.

Overall, our total count is $480 + 48 = \boxed{\textbf{(C)}~528}$.

~joshualiu315

See also

2024 AMC 10A (ProblemsAnswer KeyResources)
Preceded by
Problem 15
Followed by
Problem 17
1 2 3 4 5 6 7 8 9 10 11 12 13 14 15 16 17 18 19 20 21 22 23 24 25
All AMC 10 Problems and Solutions

The problems on this page are copyrighted by the Mathematical Association of America's American Mathematics Competitions. AMC logo.png